Health Assessment Final

Pataasin ang iyong marka sa homework at exams ngayon gamit ang Quizwiz!

The nurse holds the patient's relaxed arm with the elbow flexed at a 90 degree angle, then places the thumb over the tendon in the antecubital fossa, and strikes the thumb with the pointed part of the hammer. What deep tendon is the nurse assessing? -deltoid -biceps -brachioradialis -triceps

-biceps

You are assessing a client's eyes for the accommodation response. What would you expect to see? A. dilation of the pupils B. convergence of the axes of the eyes. C. asymmetric pupillary constriction D. tropia of the eyes.

B. convergence of the axes of the eyes. The accommodation reaction includes pupillary constriction and convergence of the axes of the eyes

A nurse on the pediatric unit is caring for a 15 year old boy with coarctation of the aorta. What is the best method to measure the blood pressure? A. in the left arm using a doppler to auscultate the systolic pressure B. in the thigh comparing the results to the blood pressure in the arm C. in the right arm palpating the radial artery first for systolic pressure D. the left arm in three positions after the boy is supine for 3 minutes

B. in the thigh comparing the results to the blood pressure in the arm

Name at least 4 risk factors for breast cancer

Age, genetic mutations, 2 or more first-degree relatives with breast cancer, post-menopausal/high endogenous estrogen or testosterone levels

Which retinal structure is normally at 3:00 and 1 disc diameter in size and 2 disc diameters temporal to the disc? A. cup B. macula C. superior retinal vessels D. inferior retinal vessels

B. macula

What muscle arises from the sternum and the medial part of the clavicle and extends diagonally across the neck to the mastoid process?

Sternocleidomastoid muscle

What lymph node is midline, behind the tip of the mandible?

Submental

A nurse assessing a 3 month old infant suspects hydrocephalus or excessive accumulation of fluid in the brain based on which finding? A. Marked asymmetry of the head B. Soft anterior fontanelle C. Increasing head circumference D. Lack of head control while sitting

C. Increasing head circumference

The NP wants to evaluate an older woman diagnosed with Parkinson's disease for her risk for falls. The nurse does a test that takes less than 10 seconds. The patient is asked to rise from an armchair, walk 10 feet, turn walk back, and sit down. What is the name of this test?

Get up and go test

Identify one of the 5 biomarkers that must exist to diagnose a client with metabolic syndrom

High BP, Increased fasting plasma glucose, elevated triglycerides, increased waist circumference, and low cholesterol

What lymph node is superficial to the mastoid process?

Posterior auricular

Which of the following eye tests best detects glaucoma or stroke, which causes an inability to see in the periphery? A. snellen eye chart exam B. confrontation C. cover test D. diagnostic positions test

B. confrontation

The practitioner is testing a male client after a head injury from a motor vehicle accident. The practitioner asks the client about how he and his wife celebrated their anniversary earlier this year. What cognitive function is the practitioner assessing?

Remote memory

What lymph node is at the base of the skull in the back of the head?

Occipital

Culture is learned by socialization and language acquisition True or False

True

How do you test the facial cranial nerve?

asking the patient to smile, frown, and puff out the cheeks

How do you test the spinal accessory cranial nerve?

assess shoulder muscle strength

A 37 year old exercise trainer states," I am really ill." He never smokes and has no history of allergies. He has no significant respiratory history. Today, he is coughing up thick white mucus and feels fatigued. His temperature is 102 degrees F orally and his pulse oximetry is 89%. He also has a headache, stuffy nose, sore throat, and chest pain when coughing. His PE reveals resonance to percussion throughout the chest, vesicular sounds in the periphery, and loud, low pitched gurgling crackles over the 4-5th ICS anteriorly and posteriorly in the bases. Which nursing diagnosis would be a priority problem? Hint: it would be an actual problem and needs to be composed correctly with a problem statement, etiology and defining characteristics.

impaired gas exchange related to excessive mucous as evidenced by crackles, SpO2 89%, and fever of 102 degrees.

How do you test the trigeminal cranial nerve?

light touch on the face with a wisp of cotton

During palpation of fremitus, what do you ask your client to say?

"Ninety-nine"

How should the nurse document mild. slightly pitting edema with no visible distortion or indentation that disappears rapidly in less than 2 seconds present in the ankles of a pregnant woman?

1+ pitting

The nurse is teaching a class on stroke and sharing with the attendees the signs to watch for in order to get to the hospital quickly. Identify the 3 changes to assess if the FAST method is used.

1. Difficulty with speech, including slurring of words. 2. Unilateral facial, arm, and/or leg weakness 3. Facial dropping.

The nurse is performing the modified Allen test. Place these steps in order. Firmly occlude both the ulnar and radial arteries in one hand while the patient makes a fist. Ask the patient to open the hand without hyperextending it. Release pressure on the ulnar artery only. Note the return to a pink color in 2-5 seconds.

1. Firmly occlude both the ulnar and radial arteries in one hand while the patient makes a fist. 2. Ask the patient to open the hand without hyperextending it. 3.Release pressure on the ulnar artery only. 4.Note the return to a pink color in 2-5 seconds.

The nurse is performing the abdominal examination. A health history has already been completed. Place the following items in their proper order.

1. Have the patient empty their bladder and lie in a supine position with the knees slightly bent. 2. Inspect the abdomen for contour, symmetry, movements,and the skin for color, moisture, temperature, edema, lesions, and hair growth. 3. Auscultate bowel sounds beginning in the RLQ with the diaphragm of the stethoscope. 4. Use the bell to auscultate for vascular sounds over the aorta, renal arteries, and femoral arteries. 5. Percuss the abdomen in all 4 quadrants 6. Palpate the abdomen first lightly then deeply in each quadrant.

The nurse is about to percuss the lungs. Place the following steps in order.

1. Hyperextend the middle finger and place only the distal joint and tip firmly against the skin. 2. Spread the fingers of the plexor hand and swish your wrist bouncing the middle finger off the stationary one. 3. Only hit the pleximeter finger just behind the nail bed with the tip of the finger and not the finger pad. 4. Repeat at least twice using staccato blows. 5. Move to the next body location and repeat this technique.

Place the steps of palpating the liver using an anterior approach in order.

1. Place your left hand under the patient's back parallel to the 11th and 12th ribs and lift up. 2. Hold your right hand on the right upper quadrant of the patient's abdomen with the fingers pointing towards the patient's head parallel to the midline. 3. Push down and under the right costal margin with your hand. 4. With each exhalation, move your hand upwards 1-2 cm.

Which nutritional assessment tool requires questions from the health care provider about everything eaten in the last 24 hours?

24 hour recall

The patient with asthma you have just admitted is in acute pain with a rating of 9 on a scale of 0-10 with 10 being unbearable chest pain. The patient also is in respiratory distress with a dyspneic rating of 10 on a scale of 0-10. The patient also has a SaO2 of 85%. The patient also does not know how to administer her bronchodilator or inhaler. As a nurse, how would you prioritize these three problems: 1=lack of knowledge, 2=pain, 3=difficulty breathing. Use the numbers to identify the problems.

3, 2, 1

Order the steps of this procedure to elicit costovertebral tenderness. 1. Thump that hand with the ulnar edge of your other fist. 2. Document if there is pain. 3. Place one hand over the 12th rib at the costovertebral angle on the back.

3. 1. 2.

A female nurse is performing a confrontation test. The nurse does the following steps in this order: 1) Positions herself at eye level 2 feet away, 2)Tells the client to look straight ahead, 3)Tells the client to cover the left eye with an opaque card while the nurse covers the left eye. 4) The nurse takes a wiggling finger and slowly advances it medially from the periphery in four different directions, superiorly, temporally, nasally and inferiorly. Which of these steps did the nurse perform incorrectly? 1,2,3, or 4 and what is the correct procedure?

3. The nurse should cover the right eye if the client is covering their left.

The nurse is taking a history for a patient experiencing tinnitus. The patient shares this problem with the nurse. What would be the next most appropriate question to ask? 1. "When was the last time you had your hearing checked?" 2. "Are you having any discharge from your ears?" 3. "Do you ever feel dizzy or lose your balance?" 4. "What medications are you currently taking?"

4. "What medications are you currently taking?"

The practitioner is assessing a client for alcohol tolerance. Which of these questions might the practitioner use to assess this condition? A. How many drinks does it take to make you feel high? B. Do you sometimes need a drink in the morning when you first get up? C. Since you have been sober, have you had any trouble sleeping or been nauseous? D. What activities have you give up as a result of drinking?

A. How many drinks does it take to make you feel high?

A 52-year-old woman has a nodule on her nose that has hard, raised, rounded lesion with pearly borders and a central red ulcer that extends deep into the dermis. She said she has noticed it for several months ago. What size would the nurse expect it to be? A. 1.5 cm B. 24 cm C. 0.75 cm D. 0.5 cm

A. 1.5 cm A nodule is a solid, elevated, hard or soft lesion larger than 1 cm., but smaller than a tumor, which is larger than a few centimeters. It may extend deep into the dermis.

The nurse palpates across the precordium. The nurse notes a vibration like a purring of a cat in the right second intercostal space near the sternal border. The nurse documents that which of these findings? A. A thrill in the aortic area. B. A murmur in the mitral area. C. A lift in the tricuspid area. D. A friction rub in the pulomonic area.

A. A thrill in the aortic area.

Which of these risk factors are associated with the development of cataracts? Select all that apply. A. African Americans B. Hispanics C. age D. Asians E. Vitamin A deficiency

A. African Americans C. age

When assessing mental status in children, which of the following statements is true? A. All aspects of mental status in children are interrelated. B. Children's mental status is impossible to assess until the child develops the ability to concentrate. C. Children are highly labile and unstable until the age of 2. D. Children's mental status is largely a function of their genetic make-up until the age of 7

A. All aspects of mental status in children are interrelated. It is difficult to separate and trace the development of just one aspect of mental status. All aspects are interdependent. For example, consciousness is rudimentary at birth because the cerebral cortex is not yet developed. The infant cannot distinguish the self from the mother's body.

The nurse is teaching a client about alcohol and its multiple effects on the body. Which of the following effects attributed to alcoholism should the nurse include in this teaching plan? Select all that apply. A. Atrial fibrillation B. Motor vehicle accidents C. Rheumatoid arthritis. D. Cardiomyopathy E. Colon cancer F. Depression G. Hypertension

A. Atrial fibrillation B. Motor vehicle accidents D. Cardiomyopathy E. Colon cancer F. Depression G. Hypertension

Which of the following are age related changes in an older person? Select all that apply. A. accumulation of body fat in the abdomen B. decreased sense of taste C. decreased gastric acid secretion D. delayed gastric emptying E. dry mouth

A. B. C. D. E. (all)

Which of the following measurement tools qualifies objectively the client's stages of withdrawal? A. CIWA B. CAGE C. SMAST-G D. AUDIT-C

A. CIWA

The nurse is teaching the football players about the symptoms of a concussion, so they will recognize this problem and seek medical attention early. Which of these symptoms should the nurse include in the list? A. Change in reaction time and coordination B. Dysphagia C. Changes in memory and judgment D. Mental cloudiness E. Loss of balance F. Nausea and vomiting

A. Change in reaction time and coordination C. Changes in memory and judgment D. Mental cloudiness E. Loss of balance F. Nausea and vomiting

Why can working with children with a different cultural perspective be especially difficult? A. Children have spiritual and cultural needs that are influenced by their stage of development. B. Parents are often the decision makers and always have no knowledge of their children's spiritual needs. C. Children at any age have spiritual and cultural needs that are a direct reflection of the culture of their peers. D. Parental perceptions about their child's illness are almost always influenced by their shaman or spiritual leader.

A. Children have spiritual and cultural needs that are influenced by their stage of development.

Which of the following are clinical manifestations that can be observed in a client diagnosed with congestive heart failure? Select all that apply A. Confusion B. Jugular venous distention C. S3 heart sound D. crackles E. Dilated pupils F. Dependent pitting edema G. Steadily decreasing oxygen saturation H. Cool moist skin I. Ascites J. Decreased ABI

A. Confusion B. Jugular venous distention C. S3 heart sound D. crackles E. Dilated pupils F. Dependent pitting edema G. Steadily decreasing oxygen saturation H. Cool moist skin I. Ascites

A 45-year-old male has come to the clinic for an abdominal assessment. Upon percussion, you note an area of dullness at the right midclavicular line starting at the sixth intercostal space. What would the nurse do? A. Consider this a normal finding and proceed with the examination. B. Document the presence of splenomegaly. C. Describe this as an enlarged massand refer him to a physician. D. Ask additional history questions regarding his alcohol intake.

A. Consider this a normal finding and proceed with the examination. The average liver span in the midclavicular line is 6-12 cm. Men and taller individuals are at the upper end of this range. Women and shorter individuals are at the lower end of this range. A liver span of 10 cm is within normal limits for this individual. The liver is located in the right upper quadrant

The nurse is palpating the breasts of a large 30 year old woman and feels a firm ridge of tissue across the bottom of both breasts in the lower quadrants. What is the next step the nurse should do? A. Document that the breasts are normal without abnormalities. B. Document that there are lumps in both breasts and include the location, size, shape, consistency, movability, distinctness, nipple location, any skin abnormalities, and tenderness, but omit any documentation of the lymph nodes. C. Document that are are lumps in both breasts and include the location, size,shape, consistency, movabililty, distinctness, nipple location, any skin abnormalities, tenderness and if there is any palpable lymph nodes. D. Document that are are lumps in both breasts and include only the distance in centimeters from the nipple and the time on the clock where the lumps are located.

A. Document that the breasts are normal without abnormalities.

Which of these perpetrators are most likely to kill a woman in this country? Select all that apply. A. Ex-husband B. Stepfather C. Next door neighbor D. Boyfriend E. Brother F. Husband

A. Ex-husband D. Boyfriend F. Husband

A physician has diagnosed a patient with stage IV ulcer. After leaving the room, a nursing student asks the nurse what the physician saw that led to that diagnosis. What should the nurse say? A. He is referring to that full thickness, irregular shaped crater on the skin extending into all skin layers. You can see the muscles and the bone inside the wound too. B. He is referring to that full thickness, irregular shaped crater on the skin extending into the subcutaneous tissue. C. He is referring to that fiery red, star-shaped marking on the cheek that has a solid circular center. D. He is referring to that blue dilation of blood vessels in a star-shaped linear pattern on the legs.

A. He is referring to that full thickness, irregular shaped crater on the skin extending into all skin layers. You can see the muscles and the bone inside the wound too.

Which of the following illustrates an appropriate category in a cultural assessment? A. Health-related Beliefs B. Chief Complaint C. Family History D. Past Medical History E. Review of Systems

A. Health-related beliefs

For the nurse, what is diagnostic reasoning? A. Helpful because it clusters and organizes cues B. Done swiftly by novice examiners C. Something that generally happens late in the nursing process D. Useless because it isn't based on evidence

A. Helpful because it clusters and organizes cues

The findings from an assessment of a 70-year-old client with swelling in his ankles includes a distended jugular venous pulse 5 cm above the sternal angle when the head of his bed is elevated 30 degrees. What does this finding indicate? A. Increased pressure in the right side of his heart B. Narrowing of the jugular veins C. Decreased fluid volume D. Increased cardiac output

A. Increased pressure in the right side of his heart Because no cardiac valve exists to separate the superior vena cava from the right atrium, the jugular veins give information about activity on the right side of the heart. They reflect filling pressures and volume changes. Normal jugular venous pulsation is 2 cm or less above the sternal angle.

A 60-year-old male is at the clinic for an eye examination. You suspect that he has a ptosis of one eye. How would you check for this? A. Inspect the distance between the palpebral fissures. B. Perform the corneal light test and look for symmetry of the light reflex. C. Perform the confrontation test. D. Assess the individual's visual acuity with the Rosenbaum chart

A. Inspect the distance between the palpebral fissures. Ptosis is drooping of the upper eyelid that would be apparent by observing the distance between the upper and lower eyelids. The upper lid should overlap the superior portion of the iris.

The nurse is preparing to perform a physical assessment. Which statement is true about the inspection phase of the physical assessment? A. Inspection takes time and reveals a surprising amount of information. B. Inspection may be somewhat uncomfortable for the expert practitioner. C. Inspection requires a quick glance at the patient's body systems before proceeding on with palpation. D. Inspection usually yields little information.

A. Inspection takes time and reveals a surprising amount of information.

The nurse is preparing to use a stethoscope for auscultation of the lungs. Which statement is true about the diaphragm? A. It is used to listen for high-pitched sounds, as breath sounds. B. It should be held lightly against the person's skin to block out low-pitched sounds. C. The nurse can hear breathe sounds through the client's gown, while maintaining privacy at the same time. D. The nurse can hear fremitus as well as normal and adventitious breath sounds.

A. It is used to listen for high-pitched sounds, as breath sounds.

To assess the dorsalis pedis artery, the nurse would palpate: A. Lateral to the extensor tendon of the great toe B. The groove behind the medial malleolus C. Over the epitrochlear lymph nodes D. Behind the knee

A. Lateral to the extensor tendon of the great toe

The nurse is preparing to auscultate for heart sounds. Which technique is correct? A. Listen by inching the stethoscope in a rough Z pattern, from the base of the heart across and down, then over to the apex B. Listen for all possible sounds for five minutes at each specified area C. Listen to the sounds at the carotid, aortic, pulmonic, and mitral areas D. Listen to the sounds only at the site where the apical pulse is palpated to be the strongest

A. Listen by inching the stethoscope in a rough Z pattern, from the base of the heart across and down, then over to the apex Do not limit auscultation of heart sounds to only 4 locations. Sounds produced by the valves may be heard all over the precordium. Inch the stethoscope in a rough Z pattern from the base of the heart across and down, then over to the apex. Or start at the apex and work your way up. Listen selectively to one sound at a time.

The nurse is preparing to auscultate for heart sounds. Which technique is correct? A. Listen by inching the stethoscope in a rough Z pattern, from the base of the heart across and down, then over to the apex. B. Listen to the sounds only at the site where the apical pulse is palpated to be the strongest. C. Listen for all possible sounds for five minutes at each specified area. D. Listen to the sounds at the carotid, aortic, pulmonic, and mitral areas.

A. Listen by inching the stethoscope in a rough Z pattern, from the base of the heart across and down, then over to the apex.

The nurse is preparing for a certification course in skin care and needs to be familiar with the various lesions that may be identified when assessing the skin. Which of the following definitions are matched correctly with the term to be used in the documentation of the lesion? Select all that apply. A. Macule: a flat and circumscribed color change of less than 1 cm. B. Vesicle: A fluid filled elevated cavity up to 1 cm in size. C. Papule: a hypertrophied scar. D. Nodule: A solid, elevated, hard or soft lesion larger than 1 cm. E. Bulla: An elevated, circumscribe lesion filled with pus.

A. Macule: a flat and circumscribed color change of less than 1 cm. B. Vesicle: A fluid filled elevated cavity up to 1 cm in size. D. Nodule: A solid, elevated, hard or soft lesion larger than 1 cm.

Which of the following are clinical manifestations of alcohol withdrawal? Select all that apply A. Nausea and vomiting B. Hypotension C. Bradycardia D. Dementia E. Hallucinations

A. Nausea and vomiting E. Hallucinations

A nurse should best use which standardized tool as a guide to assess the pain of a 65 year old client with dementia? A. PAINAD scale B. Glasgow coma scale C. CRIES scale D. FACES Revised scale

A. PAINAD Scale

Which of the following are manifestations of child abuse or neglect that warrant further assessment by the nurse and a referral to the authorities? Select all that apply A. Painful blue/grey lesions in an Asian child that are labeled by the parent as Mongolian spots and have appeared recently B. A small red mark on the back of the head of a 4 month old child that fell out of their bed according to the parents C. A large abrasion on a 10 month old child who is toddling D. Bruising and repeat hospitalizations in non-mobile children E. Fractures in the left leg that are in various stages of healing

A. Painful blue/grey lesions in an Asian child that are labeled by the parent as Mongolian spots and have appeared recently D. Bruising and repeat hospitalizations in non-mobile children E. Fractures in the left leg that are in various stages of healing

Match the assessment technique with its normal findings. A. Usually located in the 5th ICS just left of the MCL when present. B. Normal findings are up to 3 cm. above the sternal angle, which is a central venous pressure of 8 mm Hg. Document findings as "no JVD" or no jugular venous distention. C. Avoid compression when palpating and palpate only one at a time. D. Close your eyes to tune out extraneous noise and listen to all five areas first with the diaphragm then with the bell. E. A difference in the rate of the apical and radial pulses. F. A test that displaces venous blood out of the splanchnic vessels and adds volume to the venous system, thus assessing CHF.

A. Palpation of the apical pulse. B. Inspection of jugular venous pressure. C. Assessment of the carotid pulse. D. Auscultation of normal heart sounds. E. pulse deficit F. hepatojugular reflex

The nurse recognizes that which of the following is a normal physiologic change associated with the aging process? A. Peripheral blood vessels growing more rigid with age, producing a rise in systolic blood pressure. B. Hormonal changes causing vasodilation and a resulting drop in blood pressure. C. Progressive atrophy of the intramuscular calf veins, causing venous insufficiency. D. Narrowing of the inferior vena cava, causing low blood flow and increases in venous pressure resulting in varicosities.

A. Peripheral blood vessels growing more rigid with age, producing a rise in systolic blood pressure.

When using the opthalmoscope for an aged client without glasses and a past history of glaucoma, how would you proceed with the examination? A. Remove your glasses and approach the patient's left eye with your left eye. B. Move the scope around in front of the eye and ask the patient to keep his or her head still. C. Use the smaller green light and instruct the patient to focus on the opthalmoscope. D. Leave the light on in the examining room and approach the patient with the opthalmoscope anteriorly at a 90 degree angle to the face.

A. Remove your glasses and approach the patient's left eye with your left eye.

A patient has had arthritic pain in her hips for several years since a hip fracture. Her pain fluctuates from day to day. She is able to move around in her room this morning and has not offered any complaints so far this morning. However, when asked, she states that her pain is "bad this morning" and rates it at an 3 on a 1-to-10 scale. What does the nurse suspect? A. She has experienced chronic pain for years and has adapted to it. B. She is not in pain but rates it high to receive pain medication. C. She is addicted to her pain medications and cannot obtain pain relief. D. She does not want to trouble the nursing staff with her complaints.

A. She has experienced chronic pain for years and has adapted to it.

The nurse is presenting a class on risk factors for cardiovascular disease. Which of these are considered modifiable risk factors for coronary artery disease? Select all that apply A. Smoking B. Elevated BP C. Age D. Calcium deficiency E. Elevated cholesterol F. Gender G. Sedentary activity H. Race I. Family history of aortic aneurysm J. Blood sugar over 100 mg/dL K. Ethnicity

A. Smoking B. Elevated BP E. Elevated cholesterol G. Sedentary activity J. Blood sugar over 100 mg/dL

Which of these clinical manifestations would be evident in a client with deep vein thrombosis in the left leg? Select all that apply A. Swelling in the left leg of more than 2 cm B. Intermittent claudification in the left leg C. Redness in the left lower leg D. A bruit auscultated in the dorsalis pedis area E. Increased warmth in comparison to the right leg F. Dependent cyanosis

A. Swelling in the left leg of more than 2 cm C. Redness in the left lower leg E. Increased warmth in comparison to the right leg F. Dependent cyanosis

A 65 year old adult male is having postoperative pain following an abdominal colon resection. The man is rating his pain at a 9 out of 10 on a 10 point scale. Which behavioral responses should the nurse anticipate? Select all that apply. A. Tachycardia B. Increased cough C. Elevated blood pressure D. Oliguria E. Confusion F. Hypoventilation G. Fatigue H. Bradycardia I. Hyperactive bowel sounds J. Decreased cardiac output

A. Tachycardia C. Elevated blood pressure D. Oliguria E. Confusion G. Fatigue

What does an individual believe who uses a magico-religious perspective to explain the origins of their illness or disease? A. That supernatural forces control the fate of the world B. Eating imbalanced foods facilitates harmony C. Germs and viruses cause disease D. There is an imbalance within their spiritual nature

A. That supernatural forces control the fate of the world

Which of these statements regarding the lungs in the anterior chest are true? Select all that apply. A. The apex extends 3-4 cm. above the inner third of the clavicle. B. The examiner should auscultate the posterior lungs first before proceeding anteriorly. C. The lungs rest on the diaphragm at the fourth intercostal space in the midclavicular line. D. The bases are located at the sixth rib.

A. The apex extends 3-4 cm. above the inner third of the clavicle. B. The examiner should auscultate the posterior lungs first before proceeding anteriorly. D. The bases are located at the sixth rib.

The nurse is performing the diagnostic positions test. Normal findings would be best documented as which of these entries? A. The client has parallel movements in both eyes. B. The client has a slight amount of lid lag when moving from one position to another. C. The client has convergence of the eyes. D. The client has nystagmus in the extreme superior gaze.

A. The client has parallel movements in both eyes.

A client has been admitted with chronic arterial symptoms. During the assessment, the nurse should expect which findings? Select all that apply. A. The client's skin is pale and cool. B. The client states that the pain is worse at the end of the day. C. The client may state that the pain gets worse when walking. D. The nails are thickened and ridged with a flattening of the nail angle and a capillary refill of over 2 seconds. E. The client works long hours sitting at a computer desk and complains of tired, achy legs. F. The client's ankles are edematous and have weeping ulcers.

A. The client's skin is pale and cool. C. The client may state that the pain gets worse when walking. D. The nails are thickened and ridged with a flattening of the nail angle and a capillary refill of over 2 seconds.

The nurse is preparing to perform a lung assessment on an older client . The correct action by the nurse is reflected by which of these statements? A. The nurse provides explanations regarding the assessment techniques throughout the examination. B. The nurse auscultates from the apices to the bases of the lungs as the client breathes through the nose. C. The nurse follows the complete examination sequence regardless of the client 's age or condition. D. The nurse performs the examination from the left side of the bed with the client lying down.

A. The nurse provides explanations regarding the assessment techniques throughout the examination.

Many individuals embrace the hot/cold theory of health illness. Which of the following best describes the basic tenets of this theory? A. The treatment of disease consists of adding or subtracting cold, heat, dryness, or wetness to restore the balance of the humors of the body. B. Herbs and medicines are classified on their physical characteristics of hot and cold and the humors of the body. C. The four humors of the body consist of blood, yellow bile, spiritual connectedness, and social aspects of the individual. D. The causation of illness is based on supernatural forces that influence the humors of the body.

A. The treatment of disease consists of adding or subtracting cold, heat, dryness, or wetness to restore the balance of the humors of the body.

The nurse is conducting an interview. Which of these statements is true regarding open ended questions? A. They build and enhance rapport. B. They elicit feelings, opinions and ideas. C. They are used when narrative information is needed. D. They call for short one to two word answers. E. They elicit cold facts.

A. They build and enhance rapport. B. They elicit feelings, opinions and ideas. C. They are used when narrative information is needed.

In the majority culture of America, coughing, sweating, and diarrhea are symptoms of an illness. For some individuals of Mexican-American origin, however, these symptoms are a normal part of living. Why does this culture differ in their beliefs? A. They derive their meaning of health from their cultural group. B. They have more hypochondriasis than other cultures. C. They always live in lower socioeconomic areas in their community. D. They have less efficient immune systems and are often ill.

A. They derive their meaning of health from their cultural group.

Mrs. H. brings her husband to the clinic for an examination. She is particularly worried because after a recent fall, he seems to have lost a great deal of his memory of recent events. Which statement reflects your best course of action? A. You should plan to perform a mental status examination. B. You should plan to perform only a High Risk Fall Assessment on this patient. C. You should not intervene, but reassure Mrs. H that memory loss after a physical shock is normal and will subside soon. D. It would be most appropriate to refer Mr H to a psychometrician

A. You should plan to perform a mental status examination. It is necessary to perform a full mental status examination when you discover any abnormality in affect or behavior and in the following situation: family members are concerned about a person's behavioral changes (e.g., memory loss, inappropriate social interaction).

When a light is directed across the cornea of the eye from the temporal side, what should the examiner observe for if the test is abnormal? A. a scattered light reflex, which may be indicative of opacities. B. the presence of weakened eye muscles C. drainage from dacryocystitis D. the presence of conjunctivitis.

A. a scattered light reflex, which may be indicative of opacities. The presence of shadows in the anterior chamber may be a sign of acute angle-closure glaucoma. A corneal abrasion, one type of opacity, causes irregular ridges in reflected light, producing a shattered look to the light rays

A 75-year-old woman is at your clinic for a preoperative interview. This interview may take longer than interviews with younger persons. Why? A. An aged person has a longer story to tell and is likely to have multiple health concerns. B. Aged persons lose much of their mental capacities and require greater time to complete an interview. C. As aged persons mature, all are unable to hear and thus interviewers usually need to repeat much of what is said. D. All aged persons are lonely and likes to have someone to talk to.

A. an aged person has a longer story to tell and is likely to have multiple health concerns The interview usually takes longer with older adults because they have a longer story to tell.

Your client has a long history of chronic obstructive pulmonary disease. You are most likely to observe which of these findings? A. an anteroposterior : transverse diameter ratio of 1:1. B. atrophied neck and trapezius muscles. C. an anteroposterior : transverse diameter ratio of 2:1. D. increased tactile fremitus E. unequal respiratory excursion

A. an anteroposterior : transverse diameter ratio of 1:1.

Match each term used in the eye examination to its definition. A. a gray white arc commonly seen around the cornea of an elderly patient. B. protrusion of the eyeballs accompanied by retracted lids. C. structure that drains tears into the nasolacrimal ducts. D. perception of two images of a single object. E. turning in of the lower lid causing rubbing of the lashes on the cornea.

A. arcus senilus B. exopthalmos C. puncta D. diplopia E. entropion

Deep Tendon Reflex Test: The elbow is partially bent with the palm up with the thumb over the tendon. A. biceps B. triceps C. bracheoradialis D. patellar E. achilles

A. biceps

Match the appropriate heart sound to its definition. A. Abnormal heart sound that sounds like a blowing, swishing sound occurring with turbulent blood flow through the heart valves or great vessels. B. A low pitched, dull heart sound like distant thunder best heard with the bell of the stethoscope occurring immediately after closure of the semilunar valves when the atrioventricular valves open and blood pours into the ventricles. It can persist when sitting up signifying congestive heart failure. C. A blowing or swishing sound caused by turbulent blood flow heard with the bell of the stethoscope over the arteries. D. An indicator of heart failure assessed when the provider auscultates the apical pulse at the same time he or she palpates the radial pulse. pulse deficit murmur bruit S3

A. bruit B. murmur C. S3 D. pulse deficit

An African American patient is especially worried about an area of skin on her feet that has turned white. The health care provider has told her that her condition is vitiligo. Which of these descriptions would the nurse share? A. caused by the complete absence of melanin pigment. B. related to impetigo and that it can be treated with an ointment. C. caused by an excess of apocrine glands in her feet. D. caused by an excess of melanin pigment.

A. caused by the complete absence of melanin pigment. Vitiligo is the complete absence of melanin pigment in patchy areas of white or light skin on the face, neck, hands, feet, body folds, and around orifices; otherwise the depigmented skin is normal

You are assessing a client's eyes for the accommodation response. What would the examiner expect to see? A. convergence of the axes of the eyes. B. dilation of the pupils C. tropia of the eyes. D. asymmetric pupillary constriction

A. convergence of the axes of the eyes. The accommodation reaction includes pupillary constriction and convergence of the axes of the eyes

Which of the following are clinical manifestations that can be observed in a client diagnosed with congestive heart failure? Select all that apply. A. dependent pitting edema B. ascites C. crackles D. decreased ABI E. dilated pupils F. confusion G. steadily decreasing oxygen saturation H. cool moist skin I. S3 heart sound J. jugular venous distention

A. dependent pitting edema B. ascites C. crackles E. dilated pupils F. confusion G. steadily decreasing oxygen saturation H. cool moist skin I. S3 heart sound J. jugular venous distention

Match the sound with its description. A. a soft, high pitched muffled thud heard when percussing over the liver and a distended bladder B. loud high pitched,tinkling sounds heard when auscultating over the bowel indicating increased motility C. swishing or blowing sound heard when auscultating indicating constricted vessels. D. a loud, high pitched musical note heard when percussing over the stomach

A. dull B. hyperactive C. bruit D. tympany

Match the skin terms to their definitions. A. caused by increased blood flow through engorged arterioles B. a linear arrangement along a nerve root C. yellow in the sclera and mucous membranes D. an elevated cavity containing free fluid up to 1 cm E. A superficial, raised, reddened, irregular shape caused by edema

A. erythema B. zosteriform C. jaundice D. vesicle E. wheal

When evaluating a patient's pain, the nurse knows that an example of acute pain would be which of these examples? A. kidney stone B. persistent low back pain C. fibromyalgia D. osteoarthritis pain

A. kidney stone

The nurse is about to perform a retinal examination. Which light should the nurse select with the aperture selector dial to view a retina of a client with undilated pupils? A. large spot of light B. green light C. small spot of light D. narrow slit of light

A. large spot of light

The nurse notices that a patient has a solid, elevated, circumscribed lesion that is less than 0.5 cm in diameter. When documenting this finding, the nurse would report this as which of these lesions? A. papule B. bulla C. wheal D. nodule

A. papule A papule is something one can feel, is solid, elevated, circumscribed, less than 0.5 cm diameter, and is due to superficial thickening in the epidermis

Note all choices that describe a chief complaint or reason for seeking care. Select all that apply. A. statement of problem as a direct quote from patient B. treatment of problem C. symptom analysis of presenting problem D. identifies duration of problem

A. statement of problem as a direct quote from patient D. identifies duration of problem

When interviewing an adolescent, the nurse conducts a psychosocial review of symptoms. The HEADSSS method of interviewing is used. What is the purpose of this tool? A. to minimize stress B. to provide a complete picture of the client's past and present health. C. to evaluate the adolescent's health promotional practices D. to measure the adolescent's self-care abilities

A. to minimize stress

The nurse is having trouble obtaining an accurate radial pulse for an adult patient with a dysrhythmia and an irregular pulse. What would the nurse do next to obtain an accurate pulse? A. use a stethoscope and count the apical pulse for a full minute B. use a pulse oximeter checking the reading on the finger C. use a doppler checking the pulse on the feet D. use an automatic blood pressure machine, also assessing the pulse and respirations

A. use a stethoscope and count the apical pulse for a full minute

Interviewing traps include using which of these nontherapeutic techniques? A. using professional jargon B. using silence C. providing clarification D. using closed questions to gather specific data

A. using professional jargon

The nurse is teaching a class on stroke and is explaining the difference between a transient ischemic attack (TIA) and a cerebrovascular accident (CVA) or a stroke. Explain the difference.

Although both a TIA and CVA are caused by hypoxia to brain tissues, they are different because TIAs cause stroke symptoms that are temporary and last typically for an hour. TIAs are often warning signs for future CVAs or strokes, which can cause more permanent symptoms.

Avoid compression when palpating and palpate only one at a time

Assessment of the carotid pulse

Close your eyes to tune out extraneous noise and listen to all five areas first with the diaphragm then with the bell

Auscultation of normal heart sounds

Which of the following interventions constitutes "good" foot care for older persons according to the National Institute on Aging? A. Use nail polish to cover up discolored nails. B. Rotate your ankles or write the alphabet with your toes. C. Each time you take a bathe, add a generous amount of oil to the water to keep the skin soft. D. Go without shoes or slippers in the evening so your feet can breathe.

B . Rotate your ankles or write the alphabet with your toes

What is an example of subjective data? A. crepitation in the left knee B. "My left knee has been swollen and hot for the past three days." C. hemoglobin of 9.9 D. range of motion of 30 degrees of flexion in right elbow

B. "My left knee has been swollen and hot for the past three days." all others are objective.

The nurse palpates across the precordium. The nurse notes a vibration like a purring of a cat in the right second intercostal space near the sternal border. The nurse documents which of these findings? A. A murmur in the mitral area B. A thrill in the aortic area C. A friction rub in the pulmonic area D. A lift in the tricuspid area

B. A thrill in the aortic area

Among many Asians there is a belief in the yin/yang theory, rooted in the ancient Chinese philosophy of Tao. Which statement would most accurately reflect "health" in an Asian with these naturalistic beliefs? A. A person is happy, stable, and feels good. B. All aspects of the person are in perfect balance. C. A person is able to work and produce. D. A person is able to care for others and function socially.

B. All aspects of the person are in perfect balance.

Which of the following statements is true regarding assessment of the ankle-brachial index (ABI)? A. Normal ABI indices are from 0.50 to 0.80. B. An ABI 1.0 and lower indicates the presence of peripheral arterial disease and mild claudication. C. It detects increased systolic pressure distal to the area of arterial narrowing. D. The ankle pressure is usually slightly lower than the brachial pressure.

B. An ABI 1.0 and lower indicates the presence of peripheral arterial disease and mild claudication. Use of the Doppler stethoscope is a noninvasive way to determine the extent of peripheral vascular disease. The normal ankle pressure is slightly greater than or equal to the brachial pressure. An ABI of 0.80 to 1.0 indicates the presence of peripheral arterial disease and mild claudication.

Which of the following describes the best method to examine the respiratory system of the older adult male? A. Shave the chest prior to the examination of the anterior thorax. B. Ask the client when auscultating the posterior thorax to lean forward and place the arms across the lap. C. Place the client supine to allow the patient to rest. D. Allow the client to keep the gown on to keep warm.

B. Ask the client when auscultating the posterior thorax to lean forward and place the arms across the lap.

A nurse is gathering both subjective and objective data during a physical assessment and documents which of the following as subjective data? A. Respiratory rate of 18 breaths per minute. B. Client states that he is constipated. C. Apical pulse rate of 78 beats per minute D. Client has peripheral edema.

B. Client states that he is constipated.

Which of the following tests should be included when testing for hypoglossal nerve function? Select all that apply: A. ask the patient to rotate the head against the resistance of your hand B. ask the patient to stick out the tongue C. ask the patient to smile and close the eyes tightly D. ask the patient to state "light, tight and dynamite" E. ask the patient to hop on one foot

B. ask the patient to stick out the tongue E. ask the patient to hop on one foot

A fair skinned patient tells the nurse that he has noticed that one of his nevi has started to burn and bleed. When assessing his skin, the nurse would pay special attention to the danger signs for cancer and would be concerned with which additional finding? A. Diameter less than 1/8 of an inch B. Color variation C. Symmetry of lesions D.Border regularity

B. Color variation Abnormal characteristics of pigmented lesions are summarized in the mnemonic ABCD: asymmetry of pigmented lesion, border irregularity, color variation, and diameter greater than 6 mm. See slide presentation on skin.

What information is most important to remember when taking a blood pressure? A. Auscultate the pulsations of the radial artery. B. Deflate the cuff at about 2mmHg per second. C. Inflate the cuff 10 mmHg above where the brachial pulse disappears. D. Hold the arm slightly flexed at 80 degrees with the palm down.

B. Deflate the cuff at about 2mmHg per second. The pulse auscultated would be the brachial pulse, plus the arm would be supported at heart level.

Which of the following diagnostic tests best provides an objective measure of chronic or problem drinking? A. Breath alcohol analysis B. Gamma glutamyl transferase C. Bilirubin D. Mean corpuscular volume

B. Gamma glutamyl transferase

The healthcare provider is completing the health history and wants to assess if a female client has had any recent migraine headaches. Which of these questions is best to obtain this information? A. Do you ever have any headaches? B. Have you had any unusual or severe headaches in recent months? C. Do you ever have pain on one side of your head or all over? D. Do you have any history of headaches?

B. Have you had any unusual or severe headaches in recent months?

When listening to heart sounds, what does the nurse know about S1? A. It indicates the beginning of diastole B. It coincides with the carotid artery pulse C. It is louder than S2 at the base of the heart D. It is caused by closure of the semilunar valves

B. It coincides with the carotid artery pulse S1is the start of systole and is louder than S2 at the apex of the heart; S2 is louder than S1 at the base. The nurse should feel the carotid artery pulse gently while auscultating at the apex; the sound heard as each pulse is felt is S1

When listening to heart sounds, what does the nurse know about S1? A. It is caused by closure of the semilunar valves. B. It coincides with the carotid artery pulse. C. It is louder than S2 at the base of the heart. D. It indicates the beginning of diastole.

B. It coincides with the carotid artery pulse.

In assessing the carotid arteries of an older patient with cardiovascular disease, what would the nurse do? A. Palpate the artery in the upper one third of the neck and note any murmurs B. Listen with the bell of the stethoscope to assess for bruits while the patient is holding his breath on exhalation C. Instruct patient to take slow deep breaths during auscultation and listen with the diaphragm of the stethoscope D. Palpate both arteries simultaneously to compare amplitude and contour of this pulse

B. Listen with the bell of the stethoscope to assess for bruits while the patient is holding his breath on exhalation If cardiovascular disease is suspected, auscultate each carotid artery for the presence of a bruit. Avoid compressing the artery because this could create an artificial bruit and it could compromise circulation if the carotid artery is already narrowed by atherosclerosis

Which of these statements regarding movement of the extraocular muscles is true? A. Movement is controlled by neurotransmitters, as dopamine and serotonin. B. Movement is stimulated by cranial nerves III, IV, and VI. C. Movement is impaired in a patient with cataracts. D. Movement is decreased as a normal change of aging in the elderly.

B. Movement is stimulated by cranial nerves III, IV, and VI. Movement of the extraocular muscles is stimulated by three CN III, IV, and VI. p. 277

Which would be an ABNORMAL finding in the abdominal examination for a 65 year old African American male? A. An aortic width of 3 cm. B. Pain when holding the fingers under the lower liver border. C. A rounded abdomen D. Abdominal breathing E. Dullness over the right upper quadrant in the midclavicular line 1 cm below the costal margin.

B. Pain when holding the fingers under the lower liver border.

Which of the following constitutes "good" foot care for the older persons according to the National Institute on Aging? A. Use nail polish to cover up discolored nails B. Rotate your ankles or write the alphabet with your toes C. Each time you take a bath, add a generous amount of oil to the water to keep the skin soft D. Go without shoes or slippers in the evening so your feet can breathe

B. Rotate your ankles or write the alphabet with your toes

When assessing a symptom, the interviewer might organize his or her questions in this sequence: PQRSTU. The letter S in this mnemonic stands for? A. subjective data B. severity scales C. symptoms that give important clues to guide the physical exam D. social history

B. Severity scales

Select all of the following changes of aging in the thorax area. Select all that apply. A. increase in elasticity B. calcified costal cartilages with a less mobile thorax C. decreased vital capacity D. decreased residual volume E. increased number of alveoli

B. calcified costal cartilages with a less mobile thorax C. decreased vital capacity

Mrs. R. brings her 3 year old daughter into your office for a well-child visit. At the beginning of the visit, you focus your attention away from the toddler, but as the interview progresses, she begins to "warm up" and is smiling shyly at you. You will be most successful if you next do which of these actions? A. Continue to ignore her until you are ready to start the physical examination. B. Stoop down to her level and ask her about the toy she is holding. C. Tickle the toddler and get her to laugh. D. Ask Mrs. R. to leave while you examine the toddler, since toddlers often fuss less if their parents aren't in view.

B. Stoop down to her level and ask her about the toy she is holding. Very good. Although most of your communication is with the parent, do not ignore the child completely. You need to make contact to ease into the physical examination later. Begin by asking about the toys the child is playing with or about a special doll or teddy bear brought from home: "Does your doll have a name?" or "What can your truck do?" Stoop down to meet the child at his or her eye level

A client's vision is recorded as 20/30 using the Snellen eye chart. What should the nurse tell the client? A. The client can read from 30 feet what a person with normal vision can read from 20 feet. B. The client can read at 20 feet what a person with normal vision can read at 30 feet. C. The client can read the chart from 20 feet in the left eye and 30 feet in the right eye. D. At 30 feet the client can read the entire chart.

B. The client can read at 20 feet what a person with normal vision can read at 30 feet. The top number indicates the distance the person is standing from the chart; the denominator gives the distance at which a normal eye can see.

A client has been admitted with chronic arterial symptoms. During the assessment, the nurse should expect which findings? Select all that apply A. The client works long hours sitting at a computer desk and complains of tired, achy legs B. The client's skin is pale and cool C. The client's ankles are edematous and have weeping ulcers D. The client may state that the pain gets worse when walking. E. The nails are thickened and ridged with a flattening of the nail angle and a capillary refill of over 2 seconds F. The client states that the pain is worse at the end of the day

B. The client's skin is pale and cool D. The client may state that the pain gets worse when walking. E. The nails are thickened and ridged with a flattening of the nail angle and a capillary refill of over 2 seconds

An 65 year old, caucasian woman is leaving on a trip to Hawaii and has come in for a checkup. During the examination the nurse notices that she is diabetic and takes oral hypoglycemic agents. Which of these items is most important to include in her teaching plan? A. The importance of mobility and regular exercise B. The importance of sunscreen and avoiding direct sunlight C. Adequate fluid intake amounts as a result of exposure to salt water D. Prevention of orthostatic hypotension due to an increased possibility of vertigo

B. The importance of sunscreen and avoiding direct sunlight

The nurse is preparing to assess a hospitalized female patient who is experiencing significant shortness of breath. The room is warm and quiet, your hands are washed, the stethoscope is washed with alcohol, and the patient is in a gown. How should the nurse proceed with auscultation of the posterior thorax? A. The patient should lie down so that the nurse can listen with the bell of the stethoscope. B. The nurse should reach under the gown with the stethoscope to ausculate the lungs. C. The nurse should wet the stethoscope to prevent accidental crackling noises. D. The nurse should assess the lungs through the gown to ensure privacy.

B. The nurse should reach under the gown with the stethoscope to auscultate the lungs.

An 85-year-old gentleman comes to you for information on why he seems to be getting shorter as he ages. What rationale would you provide to explain that decreased height occurs with aging? A. There is a thickening of the intervertebral discs. B. There is shortening of the vertebral column. C. There is a significant loss of subcutaneous fat. D. Long bones tend to shorten with age.

B. There is shortening of the vertebral column.

You are performing an eye assessment on an 80-year-old client. Which of the following findings is considered abnormal in all elderly patients? A. Loss of the outer one third of the eyebrows B. Unequal pupillary constriction in response to light C. An increase in tear production D. Yellow or light colored small, round spots in the retinal background

B. Unequal pupillary constriction in response to light Pupillary constriction should be symmetrical. The remaining items are normal

Which of the following are possible effects on a woman's health as a results of chronic domestic violence? Select all that apply A. Less visits to a health care professional than non-battered women B. Urinary tract infections C. Sexually transmitted diseases D. Post traumatic stress disorder E. Blunt force injuries

B. Urinary tract infections C. Sexually transmitted diseases D. Post traumatic stress disorder E. Blunt force injuries

The nurse is preparing to assess the posterior thorax and lungs of a patient. Which of these statements describes correct technique for this procedure? Select all that apply. A. Use the back of the fingertips to examine skin texture, swelling, pulsation, the presence of lumps, and any lesions. B. Warm the hands first before touching the patient. C. Use the bell of the stethoscope to examine the breath sounds. D. Listen for one full respiration in each area with the stethoscope beginning with the apices and ending with the bases of the lungs, proceeding from side to side with the stethoscope. E. Confirm symmetric chest expansion by placing the hands on the anterolateral chest wall at the level of C2-T1. F. Use the fingertips to assess tactile fremitus. G. Stand behind the patient with the arms at their sides and observe the chest configuration.

B. Warm the hands first before touching the patient. D. Listen for one full respiration in each area with the stethoscope beginning with the apices and ending with the bases of the lungs, proceeding from side to side with the stethoscope. G. Stand behind the patient with the arms at their sides and observe the chest configuration.

What objective data should the nurse collect when assessing a patient's open wound? Select all that apply. A. skin turgor B. characteristics of exudate C. hair growth surrounding the wound D. width, length and depth E. color

B. characteristics of exudate D. width, length and depth E. color

During auscultation of the lungs of an adult patient, the nurse notes a coarse, loud, low pitched, bubbling sound during both inspiration and expiration. The patient also has a low pulse oximetry reading. These findings are consistent with which sound? A. pleural friction rub B. course crackles C. wheezes D. whispered pectoriloquy

B. course crackles

While performing an assessment of a 65-year-old man with a history of hypertension and coronary artery disease, the nurse notices the presence of bilateral 4+ edema in the lower legs. The skin is puffy and tight but normal in color. No increased tenderness is noted over his lower legs, and the peripheral pulses are equal and strong. In this situation, the nurse suspects that this edema is most likely caused by the following condition? A. hyperthyroidism B. heart failure C. elderly abuse D. Hepatitis

B. heart failure

To assess the dorsalis pedis artery, where would the nurse palpate? A. the groove behind the medial malleolus. B. lateral to the extensor tendon of the great toe. C. over the epitrochlear lymph nodes. D. behind the knee.

B. lateral to the extensor tendon of the great toe.

To assess the dorsalis pedis artery, where would the nurse palpate? A. over the epitrochlear lymph nodes. B. lateral to the extensor tendon of the great toe. C. the groove behind the medial malleolus. D. behind the knee.

B. lateral to the extensor tendon of the great toe. The dorsalis pedis artery is located on the dorsum of the foot. Palpate just lateral to and parallel with the extensor tendon of the big toe.

The nurse just noted from a patient's medical record that the patient has a lesion that is confluent in nature. On examination, what would the nurse expect to find? A. lesions arranged in a line along a nerve route. B. lesions that run together. C. lesions that are annular and clustered together. D. lesions that are twisted and snakelike.

B. lesions that run together. Grouped lesions are clustered together. Polycyclic lesions are annular in nature. Zosteriform lesions are arranged along a nerve route. Confluent lesions run together.

Illness is seen as a part of life's rhythmic course and as an outward sign of disharmony within. Which term most accurately reflects this view of illness? A. magico-religious theory B. naturalistic theory C. biomedical theory D. reductionist theory

B. naturalistic theory

The nurse is examining a 34 year old client who is having a postoperative checkup in her home. This person states, she is nearsighted (myopia) in both eyes. The nurse's eyes are normal. When examining the eyes, the nurse begins at 0 diopters and adjusts the diopters in the following manner? A. positively using the black numbers B. negatively using the red numbers

B. negatively using the red numbers

A patient, who is malnourished, comes to the clinic and tells the nurse that he has been confined to his recliner chair for about 3 days and he wants the nurse to evaluate his sacrum. During the assessment, what would the nurse expect to find? A. blanchable red, mushy, cool skin over the bony prominences B. non-blanchable red, firm, warm skin over the bony prominences C. pale keloid with a decreased capillary filling time D. cool skin with cyanosis over the sacrum

B. non-blanchable red, firm, warm skin over the bony prominences

The nurse will use which technique of assessment to determine the presence of crepitus, swelling, and pulsations? A. Percussion B. Palpation C. Inspection D. Auscultation

B. palpation

The nurse is examining a 34 year old client who is having a postoperative checkup in her home. This person states, she is farsighted (hypermyopia) in both eyes. The nurse's eyes are normal. When examining the eyes, the nurse begins at 0 diopters and adjusts the diopters in the following manner? A. negatively using the red numbers B. positively using the black numbers

B. positively using the black numbers

The patient is given a coin in her hand and does not recognize it. What is this test called? A. psoas sign B. stereognosis C. ballottement test D. graphesthesia E. McMurray's test

B. stereognosis

Which of the following is considered an example of objective data? A. complaints of dizziness B. swollen, tender lymph nodes C. a history of chronic sore throats D. a patient description of an earache

B. swollen, tender lymph nodes

The nurse is conducting an interview with Jeremy, a 17 year old male accompanied by his mother. Which statement by the nurse is most therapeutic and will encourage the teen to express himself when conducting the sexual history? A. "Mrs. Williams,does Jeremy have a history of STD's?" B. Tell me about your sex life. C. "Mrs. Williams, is your son sexually active?" D. "Are you sexually active yet?"

B. tell me about your sex life

Deep Tendon Reflex Test: The arm is hung freely as it is supported by the nondominant hand. A. biceps B. triceps C. bracheoradialis D. patellar E. achilles

B. triceps

You are assigned to take complete health histories on all the patients attending a wellness workshop. As a part of this history, you ask, "You don't smoke, drink, or take drugs do you?" What is this an example of? A. using blunt language to deal with distasteful topics. B. using biased or leading questions. C. talking too much. D. using confrontation

B. using biased and leading questions

When auscultating the lungs of an adult client, you note that over the posterior lower lobes you hear low-pitched, soft breath sounds, with inspiration being longer than expiration. You know that these are which sounds? A. bronchovesicular breathe sounds and these are normal in that location B. vesicular breathe sounds and these are normal in that location C. bronchial breath sounds and these are normal in that location D. bronchophony sounds and these are normal in that location

B. vesicular breathe sounds and these are normal in that location Vesicular breath sounds are low-pitched, soft sounds with inspiration being longer than expiration. These breath sounds are expected over peripheral lung fields where air flows through smaller bronchioles and alveoli.

Which anthropometric measurement is an indicator of obesity and calculated with the client's height and weight?

BMI

A blowing or swishing sound caused by turbulent blood flow heard with the bell of the stethoscope over the arteries

Bruit

The nurse is teaching a 50 year old woman about the benefits of mammography. Which of these statements by the patient would indicate that further teaching is necessary? A. "I read that mammograms should be done annually for women my age." B. "I read that mammograms are recommended for screening for breast cancer and should start at age 40." C. "I read that mammograms take the place of self-breast examinations." D. "I read that mammograms reveal small lumps to small to be detected by self-breast examination."

C. "I read that mammograms take the place of self-breast examinations."

Which statement indicates that the nurse understands the pain experienced by an older adult? A. "Older individuals perceive pain to a lesser degree than do younger individuals." B. "Older adults must learn to tolerate pain." C. "Pain indicates a pathologic condition or an injury and is not a normal process of aging." D. "Pain is a normal process of aging and is to be expected."

C. "Pain indicates a pathologic condition or an injury and is not a normal process of aging."

Which of these entries is best to document a normal carotid pulse? A. The carotid is within normal limits. B. 3+ carotids C. 2+ carotids without bruits bilaterally D. Carotids present on both sides.

C. 2+ carotids without bruits bilaterally

You are having trouble auscultating the client's bowel sounds. How long should you auscultate before charting that bowel sounds are absent? A. 5 seconds B. 10 minutes C. 5 minutes D. 15 minutes

C. 5 minutes

The nurse knows that which statement is true regarding the pain experienced by infants? A. Infants feel pain less than do adults. B. The FPS-R can be used to assess pain in infants. C. A procedure that induces pain in adults will also induce pain in the infant. D. Pain in infants can only be assessed by physiologic changes, such as an increased heart rate.

C. A procedure that induces pain in adults will also induce pain in the infant.

Which of the following statements is true regarding assessment of the ankle-brachial index (ABI)? A. It detects increased systolic pressure distal to the area of arterial narrowing B. Normal ABI indices are from 0.50-1.0 C. An ABI of 0.80-0.90 indicates the presence of peripheral arterial disease D. The ankle pressure is usually slightly lower than the brachial pressure

C. An ABI of 0.80-0.90 indicates the presence of peripheral arterial disease Use of the Doppler stethoscope is a noninvasive way to determine the extent of peripheral vascular disease. The normal ankle pressure is slightly greater than or equal to the brachial pressure. An ABI of 0.80-1.0 indicates the presence of peripheral arterial disease and mild claudication.

Which of the following assessment findings is most consistent with clubbing of the fingernails? A. A nail base that is firm to palpation and slightly tender. B. Curved nails with a convex profile and ridges across the nail. C. An angle of the nail base of 180 degrees or greater with a nail base that feels spongy. D. A nail base that feels spongy with an angle of the nail base of 120 degrees.

C. An angle of the nail base of 180 degrees or greater with a nail base that feels spongy. The normal nail is firm at its base and has an angle of 160 degrees. In clubbing, the angle straightens to 180 degrees or greater and the nail base feels spongy.

In the hot/cold theory, illnesses are believed to be caused by hot or cold entering the body. Which of the following patients' conditions is most consistent with a "cold" condition? A. A diabetic patient with renal failure. B. A teenager with an abscessed tooth C. An elderly male with gastrointestinal discomfort D. A child with symptoms of itching and a rash

C. An elderly male with gastrointestinal discomfort

A residential male college student comes to the clinic with complaints of constant excruciating headaches. The cerebrospinal analysis confirms a diagnosis of bacterial meningitis. Which of these questions should the nurse ask to further validate this diagnosis? A. Are any of your lymph nodes in your neck tender? B. Are you having any dizziness? C. Are you having any neck pain and/or stiffness? D. Have you had any change in mood or energy levels recently?

C. Are you having any neck pain and/or stiffness?

When examining the thyroid gland with a posterior approach on the left side, the examiner first... then... Put the steps in the correct order A. Use your fingers to palpate the gland as the patient swallows B. Repeat these steps for the right side C. Ask the patient to bend the head slightly forward and to the left D. Auscultate the gland if enlarged E. Use the fingers of your right hand to push the trachea slightly to the left

C. Ask the patient to bend the head slightly forward and to the left E. Use the fingers of your right hand to push the trachea slightly to the left A. Use your fingers to palpate the gland as the patient swallows B. Repeat these steps for the right side D. Auscultate the gland if enlarged

A client has suddenly developed shortness of breath and appears to be in significant respiratory distress. After placing the client on oxygen but before putting in a call to the physician, which of these actions is best for the nurse to take when assessing the client further? A. Assess the client's pulse and blood pressure. B. Call for a chest x-ray and wait for the results before beginning an assessment. C. Auscultate the thorax bilaterally, noting any differences in percussion tones and lung sounds. D. Inspect the thorax for any new masses and bleeding associated with respirations.

C. Auscultate the thorax bilaterally, noting any differences in percussion tones and lung sounds.

How best can the nurse determine stroke volume? A. Measure the strength of the radial pulse. B. Take the blood pressure while the client is standing. C. Calculate the difference between the systolic and the diastolic blood pressure. D. Subtract the difference between the radial and apical pulse

C. Calculate the difference between the systolic and the diastolic blood pressure.

Which technique is most correct to use when assessing the pupillary light reflex? A. Ensure that the lights are on, then ask the patient to follow your penlight in eight directions and observe for bilateral pupil constriction. B. Darken the room then ask the patient to gaze at your nose. Then shine a penlight directly in front of the patient and inspect for pupillary constriction. C. Darken the room then ask the patient to gaze into the distance. Then shine a light advancing from the side of the pupil and observe for direct and consensual pupillary constriction. D. Ask the patient to focus on a distant object. Then ask the patient to follow your penlight to about 7 cm from the nose, then shine the light into the pupils.

C. Darken the room then ask the patient to gaze into the distance. Then shine a light advancing from the side of the pupil and observe for direct and consensual pupillary constriction. To test the pupillary light reflex, advance a light in from the side and note the direct and consensual pupillary constriction

How should the fontanels feel when palpated in the calm and happy infant? A. Pulsating B. Tense or bulging C. Firm, slightly concave, and well defined D. Depressed or sunken

C. Firm, slightly concave, and well defined

The nurse is performing the modified Allen test. Place these steps in order. A. Ask the patient to open the hand without hyperextending it. B. Note the return to a pink color in 2-5 seconds. C. Firmly occlude both the ulnar and radial arteries in one hand while the patient makes a fist. D. Release pressure on the ulnar artery only.

C. Firmly occlude both the ulnar and radial arteries in one hand while the patient makes a fist. A. Ask the patient to open the hand without hyperextending it. D. Release pressure on the ulnar artery only. B. Note the return to a pink color in 2-5 seconds.

A 65 year old woman is asked by her daughter to see the NP and have a mental status examination. The practitioner shares with her a list of words and asks her to recall them 5 minutes later. To test the duration of her memory, when should the practitioner ask the woman to repeat the words again? A. In 20 minutes B. In 60 minutes C. In 10 minutes D. In 7 minutes

C. In 10 minutes

The nurse has discovered decreased skin turgor in a patient and knows that this is an expected finding in which of the following conditions? A. With conditions of connective tissue disorders such as scleroderma B. Cases of severe obesity C. In an individual who is severely dehydrated D. During childhood growth spurts

C. In an individual who is severely dehydrated Decreased skin turgor is associated with severe dehydration, aging, or extreme weight loss

The nurse has discovered decreased skin turgor in a patient and knows that this is an expected finding in a dehydrated patient. Where is it best for the nurse to assess turgor? A. In the second intercostal space just left of the sternum B.On the abdomen 2 inches from the umbilicus C.On the anterior chest under the clavicle D.On the wrist adjacent to the radial pulse

C. On the anterior chest under the clavicle Decreased skin turgor is associated with severe dehydration, aging, or extreme weight loss

The nurse recognizes that which of the following is a normal physiologic change associated with the aging process? A. Narrowing of the inferior vena cava, causing low blood flow and increases in venous pressure resulting in varicosities. B. Progressive atrophy of the intramuscular calf veins, causing venous insufficiency C. Peripheral blood vessels growing more rigid with age, producing a rise in systolic blood pressure D. Hormonal changes causing vasodilation and a resulting drop in blood pressure.

C. Peripheral blood vessels growing more rigid with age, producing a rise in systolic blood pressure

A teenage girl has arrived complaining of pain in her left wrist. She has been playing basketball and fell, landing on her left hand. You examine her hand, and expect a fracture if which of these conditions existed? A. She complains of dull throbbing pain that increases with rest. B. She complains of a dull ache. C. She complains of sharp pain that increases with movement. D. She complains that the pain in her wrist is deep.

C. She complains of sharp pain that increases with movement.

The client following a head injury is difficult to arouse. The nurse is only able to get the client to respond by rubbing the sternum causing pain. In response, the client groans and becomes restless. What level of consciousness would the nurse record for this client? A. Comatose B. Obtunded C. Stuporous D. Lethargic

C. Stuporous

The nurse is reviewing percussion techniques with a newly graduated nurse. Which technique, if used by the new nurse, indicates that more review is needed? A. The nurse uses the wrist to make the strikes, not the forearm. B. The nurse strikes with the tip of the middle finger. C. The nurse leaves the striking finger on the pleximeter or the middle finger of the nondominant hand after each stroke. D. The nurse percusses two times over each area.

C. The nurse leaves the striking finger on the pleximeter or the middle finger of the nondominant hand after each stroke.

Which of the following is a normal finding in the abdominal assessment? A. The presence of a bruit in the femoral area. B. A dull percussion note in the left upper quadrant as the patient is supine. C. The patient feels a thud when the nurse indirectly percusses the costal vertebral angle. D. Hyperactive bowel sounds in all 4 quadrants.

C. The patient feels a thud when the nurse indirectly percusses the costal vertebral angle. Tympany should predominate in all four quadrants of the abdomen because air in the intestines rises to the surface when the person is supine. The LUQ would be typanic also. A bruit would indicate atherosclerosis in the blood vessel.

Note which of these statements are true regarding the abdominal assessment. Select all that apply. A. Percussion of the aorta is always done. B. Auscultation over the stomach is the best method of determining if the nasogastric feeding tube is in the proper place. C. The patient should lie supine with their arms at their sides. D. The normal percussion sound in the abdominal area is tympany. E. The abdomen should be symmetric.

C. The patient should lie supine with their arms at their sides. D. The normal percussion sound in the abdominal area is tympany. E. The abdomen should be symmetric.

An 85-year-old gentleman comes to you for information on why he seems to be having trouble smelling his food as he ages. Which of these changes occurs with aging that would cause his problem? A. Nerves tend to shorten with age. B. There is shortening of the vertebral column. C. There is a significant delay in transmission over the synapses. D. There is a thickening of the intervertebral discs.

C. There is a significant delay in transmission over the synapses.

The nurse is preparing to conduct a health history. Which of these statements best describes the purpose of a health history? A. To provide data concerning the patient's communication patterns. B. To provide a database of objective information about the patient. C. To make a diagnosis about the health status of the individual D. To provide an opportunity for interaction between the patient and the nurse.

C. To make a diagnosis about the health status of the individual The purpose of the health history is to collect subjective (not objective) data—what the person says about himself or herself and as a basis for the PE to arrive at a diagnosis

The nurse is assessing a patient's pain. What does the nurse know to be the most reliable indicator of pain? A. Patient's vital signs B. Physical examination C. Subjective report D. Results of a computerized axial tomographic scan

C. subjective report

Which of these areas of the client's history helps to organize and illustrate the family's health problems? A. the biographical data B. the relationships within the family C. the genogram D. the review of symptoms

C. the genogram

The nurse is preparing to assess the posterior thorax and lungs of a patient. Which of these statements describe the correct technique for this procedure? Select all that apply. A. Use the bell of the stethoscope to examine the breath sounds. B. Use the back of the fingertips to examine skin texture, swelling, pulsation, the presence of lumps, and any lesions. C. Warm the hands first before touching the patient. D. Use the fingertips to assess tactile fremitus. E. Confirm symmetric chest expansion by placing the hands on the anterolateral chest wall at the level of C2-T1. F. Listen for one full respiration in each area with the stethoscope beginning with the apices and ending with the bases of the lungs, proceeding from side to side with the stethoscope. G. Stand behind the client with the arms at their sides and observe the chest configuration.

C. Warm the hands first before touching the patient. F. Listen for one full respiration in each area with the stethoscope beginning with the apices and ending with the bases of the lungs, proceeding from side to side with the stethoscope. G. Stand behind the client with the arms at their sides and observe the chest configuration.

The nurse is reviewing an assessment of client's peripheral pulses and notes that the documentation states that the radial pulses are both 1+. The nurse recognizes that this reading indicates what type of pulse? A. Bounding B. Normal C. Weak D. Absent

C. Weak

The nurse is preparing to complete a health assessment on a 16-year-old girl whose parents have brought her to the clinic. Which instruction would be appropriate for the parents before the interview begins? A. Please stay during the interview, you can answer for her if she does not know the answer. B. It would help to interview the three of you together. C. While I interview your daughter, will you step out to the waiting room and complete these family health history questionnaires. D. While I interview your daughter, will you please stay in the room and complete these family health history questionnaires.

C. While I interview your daughter, will you step out to the waiting room and complete these family health history questionnaires Interview the girl alone. The parents can wait outside and fill out family health history questionnaires

The examiner provides resistance as the client shrugs the shoulders. This is a test of which cranial nerve? A. II B. IX C. XI D. V

C. XI

Deep Tendon Reflex Test: The radius is tapped with the flat side of the hammer. A. biceps B. triceps C. bracheoradialis D. patellar E. achilles

C. bracheoradialis

During an interview, the nurse states, "You mentioned shortness of breath. When did this start?" Which verbal skill is used with this statement? A. open ended questioning B. reflection C. closed ended questioning D. facilitation

C. closed ended questioning

A 45-year-old male has come to the clinic for an abdominal assessment. Upon percussion, you note an area of dullness at the right midclavicular line 1 centimeter above the costal margin. What would you do? A. describe this as an enlarged liver and refer him to a physician. B. document the presence of hepatomegaly C. consider this a normal finding and proceed with the examination. D. ask additional history questions regarding his alcohol intake.

C. consider this a normal finding and proceed with the examination. The average liver span in the midclavicular line is 6-12 cm. Men and taller individuals are at the upper end of this range. Women and shorter individuals are at the lower end of this range. A liver 2 cm below the costal margin is within normal limits for this individual.

During auscultation of the lungs of an adult client , the nurse notes a coarse, loud, low pitched, bubbling sound during both inspiration and expiration. The client also has a low pulse oximetry reading. These findings are consistent with which sound? A. wheezes B. whispered pectoriloquy C. course crackles D. pleural friction rub

C. course crackles

Which of these clinical manifestations would be evident in a client with deep vein thrombosis in the left leg? Select all that apply. A. a bruit auscultated in the dorsalis pedis area B. intermittent claudification in the left leg C. dependent cyanosis D. redness in the leg lower leg E. swelling in the left leg of more than 2 cm F. increased warmth in comparison to the right leg

C. dependent cyanosis D. redness in the leg lower leg E. swelling in the left leg of more than 2 cm F. increased warmth in comparison to the right leg

Which of these instruments is used to measure angles of joint flexion? A. protractor B. calipers C. goniometer D. reflex hammer

C. goniometer

he findings from an assessment of a 70-year-old client with swelling in his ankles include a distended jugular venous pulse 5 cm above the sternal angle when the head of his bed is elevated 30 degrees. What does this finding indicate? A. narrowing of jugular veins. B. increased cardiac output C. increased pressure in the right side of his heart. D. decreased fluid volume

C. increased pressure in the right side of his heart.

The nurse is preparing to use a stethoscope for auscultation. Which statement is true regarding the diaphragm of the stethoscope? A. It is used to listen to low pitched sounds. B. It should be cold when placing it on the patient's skin to induce dilation of the blood vessels. C. It is used to listen for high pitched sounds. D. It should be held lightly against the body part being auscultated.

C. it is used to listen to high pitched sounds

In assessing the carotid arteries of an older patient with cardiovascular disease, what would the nurse do? A. palpate both arteries simultaneously to compare amplitude and contour of this pulse. B. palpate the artery in the upper one third of the neck and note any murmurs. C. listen with the bell of the stethoscope to assess for bruits while the patient is holding his breathe on exhilation. D. instruct patient to take slow deep breaths during auscultation and listen with the diaphragm of the stethoscope.

C. listen with the bell of the stethoscope to assess for bruits while the patient is holding his breathe on exhilation.

When examining the optic disc, the nurse notes a creamy yellow-orange disc with arteries, brighter and redder than the veins, crossing the disc that are about 1/3 more narrow than the veins. How does the nurse interpret these findings? A. indicates drusen bodies B. suggests diabetic retinopathy C. normal findings D. suggests hypertensive retinopathy

C. normal findings

Striae may be observed when the elastic fibers in the reticular layer of the skin are broken following rapid or prolonged stretching as during pregnancy. What is the color of new striae? A. black B. gray C. pink D. silvery white E. yellow

C. pink

The nurse is preparing to percuss the patient's lungs. What is the purpose of percussion? A. to assess the symmetry and consistency of the underlying organs. B. to assess the body's turgor C. to assess the density of the underlying tissues. D. to accurately assess the texture of the organs

C. to assess the density of the underlying tissues. Percussion yields a sound that depicts the location, size, and density of the underlying organ. Turgor and texture are assessed with palpation

What is the main reason auscultation precedes percussion and palpation of the abdomen? A. to determine areas of tenderness before employing percussion and palpation. B. to prevent distortion of vascular sounds such as bruits and hums that might occur after percussion and palpation. C. to prevent distortion of bowel sounds that might occur after percussion and palpation. D. to allow the patient more time to relax and, therefore, be more comfortable with the physical examination.

C. to prevent distortion of bowel sounds that might occur after percussion and palpation. This is done because percussion and palpation can increase peristalsis, which would give a false interpretation of bowel sounds

During an examination, the nurse finds that a patient has excess dryness of the skin. What is the best term to describe this condition? A. scoliosis. B. seborritus. C. xerosis. D. pruritus.

C. xerosis. Xerosis is the term used to describe skin that is excessively dry

The nurse is caring for a 4 year old Asian American who can neither read or speak English. What would be the appropriate eye chart to use to assess the client's vision? A. Jaeger chart B. Allen C. Ishihara chart D. Snellen E chart E. Wong Faces scale

D. Snellen E chart

True or False: The best position for the nurse to auscultate S3 and S4 heart sounds is in a supine position with the head of the bed in semi-fowlers.

False

A 23-year-old patient in the clinic appears anxious. Her speech is rapid. She is fidgety and in constant motion. Which of the following questions or statements would be most appropriate to use in this situation to assess attention span? A. "Please describe the meaning of the phrase, 'Looking through rose-colored glasses.'" B. "How do you usually feel? Is this normal behavior for you?" C. "I am going to say four words. In a few minutes, I will ask you to recall them." D. "Please pick up the pencil in your left hand, move it to your right hand, and place it on the table."

D. "Please pick up the pencil in your left hand, move it to your right hand, and place it on the table." Attention span is evaluated by assessing the individual's ability to concentrate and complete a thought or task without wandering. Giving a series of directions to follow is one method used to assess attention span.

When providing culturally competent care, nurses must incorporate cultural assessment into their health assessment. Which statement is most appropriate to use when initiating an assessment of cultural beliefs with an elderly Native American patient? A. "Are you of the Christian faith?" B. "Do you want to see a medicine man?" C. "How often do you seek help from medical providers?" D. "What cultural or spiritual beliefs are important to you?"

D. "What cultural or spiritual beliefs are important to you?"

True or False: The patient has a number drawn in the palm of her hand and is asked to identify the number. This test is called clonus.

False

During inspection of the precordium of an adult client with a thin chest wall, the nurse notices the chest visibly moving upward at the 4th intercostal space within the midclavicular line. What does this finding most likely suggest? A. Enlargement of the right ventricle B. Stenosis of the aortic valve C. Right ventricular heave D. A normal heart

D. A normal heart

Which of the following statements is true regarding assessment of the ankle-brachial index (ABI?) A. The ankle pressure is usually slightly lower than the brachial pressure. B. It detects increased systolic pressure distal to the area of arterial narrowing. C. Normal ABI indices are from 0.50 to 1.0. D. An ABI of 0.80-.90 indicates the presence of peripheral arterial disease.

D. An ABI of 0.80-.90 indicates the presence of peripheral arterial disease.

Older persons are at risk for glaucoma and should receive annual eye examinations. Which of these questions should the nurse best ask to screen a person for glaucoma? A. Do you smoke? How many packs per day? B. Do your eyes feel dry and burning in recent weeks? C. Are you having any progressive blurring of your vision? D. Are you experiencing any loss of peripheral vision?

D. Are you experiencing any loss of peripheral vision?

When assessing a client, the nurse documents the left dorsalis pedis pulse as 2+ on a 0-4+ scale. What assessment would the nurse do next? A. Left posterior tibial pulse B. Left femoral pulse C. Left aortic pulse D. Do nothing as this is a normal assessment

D. Do nothing as this is a normal assessment

When assessing a client, the nurse documents the left dorsalis pedis pulse as 2+ on a 0-4+ scale. What assessment would the nurse do next? A. left posterior tibial pulse B. left aortic pulse C. left femoral pulse D. Do nothing as this is a normal assessment

D. Do nothing as this is a normal assessment

A 70 year old man has been increasingly forgetful. The practitioner at the clinic performs a mini-cog assessment. Which of these directions would be appropriate when conducing this assessment? A. Write todays date on this blank sheet of paper. B. Write down how do you feel today in one word. C. Repeat 4 words back to me after I say them. D. Draw a clock on a blank piece of paper

D. Draw a clock on a blank piece of paper

The nurse is examining the client who has cellulitis in the hand. The lymph nodes in the humerus above the medial condyle are 2mm in size, discrete, hard, tender, fixed, and palpable. Which nodes does the nurse document are abnormal? A. Superficial cervical B. Inguinal C. Supracalvicular D. Epitrochlear

D. Epitrochlear

The nurse is examining the client who has cellulitis in the hand. The lymph nodes in the humerus above the medial condyle are 2mm in size, discrete,hard, tender, fixed, and palpable. Which nodes does the nurse document are abnormal? A. supraclavicular B. superficial cervical C. inguinal D. epitrochlear

D. Epitrochlear

What assessment will assist in keeping the client safe who is taking antihypertensive agents? A. Noting a widening pulse pressure. B. Asking whether the client is light headed. C. Assessing for a rise in blood pressure when standing. D. Evaluating orthostatic hypotension.

D. Evaluating orthostatic hypotension.

A physician in the clinic is frequently bothered when clients arrive late for appointments. He is so irritated by this behavior that he often finds it hard to provide appropriate care to these individuals. What should he do FIRST in trying to overcome his difficulty? A. Allow the patients to arrive late and build this into his schedule. B. Understand that these cultural practices are helpful to the patient. C. Identify the meaning of health to the patient. D. Examine his own culturally based values, beliefs, attitudes, and practices.

D. Examine his own culturally based values, beliefs, attitudes, and practices.

A nurse is assessing a 4 year old child that states she has a owie in her head after falling from the slide in the playground. She points to her forehead. What tool would be best to use to assess her pain accuragtely? A. McGill Pain Questionnaire B. The Analogue scale C. The FLACC scale D. FACES Revised

D. FACES revised

A Native American woman has come to your clinic to seek help with regulating her diabetes. Based on your knowledge of this culture, what changes to the care plan would you make? A. Provide extra help in dealing with her illness for she may be experiencing a crisis of faith. B. Have her give up her beliefs of the naturalistic causes of disease. C. Make her always comply with the treatment prescribed. D. Incorporate the assistance of a shaman or medicine man.

D. Incorporate the assistance of a shaman or medicine man.

The nurse is conducting a DEVELOPMENTAL history on a 5 year old child. Which questions are appropriate to ask the parents for this part of the assessment? Select all that apply. A. How much junk food does your child eat? B. Does he take a children's vitamin? C. Does he have any food allergies? D. Is he able to tie his shoelaces? E. How many teeth has he lost, and when did he lose them? F. Can he tell time?

D. Is he able to tie his shoelaces? E. How many teeth has he lost, and when did he lose them? F. Can he tell time? Questions about tooth loss, ability to tell time, and ability to tie shoelaces are appropriate questions for a developmental assessment. Questions about junk food intake and vitamins are part of a nutritional history. Questions about food allergies are not part of a developmental history

The nurse is testing visual accommodation. This term refers to which of these actions? A. Changes in peripheral vision when a wiggling finger is advanced from the side. B. Pupillary dilation when looking at a far object. C. Involuntary blinking in the presence of bright light. D. Pupillary constriction when looking at a near object.

D. Pupillary constriction when looking at a near object.

A teenage girl has arrived for a scoliosis screening. You would suspect this condition during the screening if which of these conditions were present? A. She has kyphosis when standing straight. B. She complains of dull throbbing pain that increases with movement. C. She complains of a dull ache when she bends forward. D. She has asymmetry of the shoulders, scapulae, and hips.

D. She has asymmetry of the shoulders, scapulae, and hips.

The nurse examines an infant with positional plagiocephaly. What should the nurse share as to the probable cause of this molding? A. A buildup of cerebrospinal fluid in the head causing it to enlarge B. Hemorrhaging that will resorb in the first few days of the infant's life. C. Birth trauma that will resolve in the first few days of life. D. Sleeping on the back to prevent sudden infant death syndrome

D. Sleeping on the back to prevent sudden infant death syndrome

What would the nurse expect when taking the blood pressure of a client who is taking blood thinners, such as warfarin? A. The blood pressure would be increased because of increased arterial resistance. B. The increased cardiac output would increase the blood pressure. C. The elasticity would be decreased as well as the blood pressure. D. The blood viscosity would be thinner causing the blood pressure to decrease.

D. The blood viscosity would be thinner causing the blood pressure to decrease.

To assess the head control of a 4-month-old, you lift him up in a sitting position while holding the wrists. What should the infant's response be? A. The arms will extend and the head drop down. B. The knees and elbows will extend with the back curved in a S. C. The toes will fan out on both feet. D. The head will lift up and not flop forward.

D. The head will lift up and not flop forward.

The nurse is reviewing an assessment of client's peripheral pulses and notes that the documentation states that the radial pulses are both: 1+. The nurse recognizes that this reading indicates what type of pulse? A. Absent B. Normal C. Bounding D. Weak

D. Weak

After the health history, the nurse asks the client to define his perception of health. Which of the following questions is appropriate for this portion of the health history? A. How often do you perform your testicular self-examination? B. What do you think caused your problem? C. How do you spend a typical day? D. What do you expect from us as health care providers?

D. What do you expect from us as health care providers?

During inspection of the precordium of an adult client with a thin chest wall, the nurse notices the chest visibly moving upward at the 4th intercostal space within the midclavicular line. What does this finding most likely suggest? A. stenosis of the aortic valve B. right ventricular heave C. enlargement of the right ventricle D. a normal heart

D. a normal heart

A 30-year-old woman has recently moved to the United States with her husband. They are living with relatives until they can get a house of their own. When company arrives to visit with her sister, she feels suddenly shy and retreats to the back bedroom to hide until the company leaves again. She states that this is just because she doesn't know how to speak perfect English and does not know the social customs of the US. What could this woman be experiencing? A. cultural disorientation B. ethnocentricity C. cultural taboos D. acculturative stress E. cultural unfamiliarity

D. acculturative stress

The nurse is bathing an 80-year-old man and notices that his skin is wrinkled, thin, lax, and dry. What would this finding be related to? A. an increase in elastic fibers and collagen and a decrease in subcutaneous fat in the elderly. B. increased numbers of sweat and sebaceous glands in the elderly. C. increased vascularity of the skin in the elderly. D. an increased loss of elastin and collagen and a decrease in subcutaneous fat in the elderly.

D. an increased loss of elastin and collagen and a decrease in subcutaneous fat in the elderly. An accumulation of factors place the aging person at risk for skin disease and breakdown: the thinning of the skin, the decrease in vascularity and nutrients, the loss of protective cushioning of the subcutaneous layer, a lifetime of environmental trauma to skin, the social changes of aging, the increasingly sedentary lifestyle, and the chance of immobility

During an examination of an adult male that has a medical diagnosis of left lower lobe pneumonia, the nurse recognizes that egophony would most likely sound like which of these descriptions? A. clear and distinct B. faint and muffled and almost inaudible C. eeeeeeeeeeeee D. bleating long aaaaaaaaa

D. bleating long aaaaaaaaa

The nurse is assessing a patient's skin during an office visit. What part of the hand and technique should be used to best assess the patient's skin temperature? A. Ulnar portion of the hand; increased blood supply in this area enhances temperature sensitivity. B. Fingertips; they are more sensitive to small changes in temperature C. Palmar surface of the hand; this surface is the most sensitive to temperature variations because of its increased nerve supply in this area. D. Dorsal surface of the hand; the skin is thinner on this surface than on the palms

D. dorsal surface of the hand; the skin is thinner on this surface than the palm

The nurse is presenting a class on risk factors for cardiovascular disease. Which of these are considered modifiable risk factors for coronary artery disease (CAD)? Select all that apply. A. ethnicity B. gender C. race D. elevated cholesterol E. age F. family history of aortic aneurysm G. calcium deficiency H. sedentary activity I. blood sugar over 100 mg/dl J. smoking K. elevated blood pressure

D. elevated cholesterol H. sedentary activity I. blood sugar over 100 mg/dl J. smoking K. elevated blood pressure

If a Hispanic person is experiencing a symptom, such as high blood sugar due to diabetes (which is felt to be "hot"), what are they most likely to treat it with? A. no treatment at all, since illness is an expected part of life. B. readings and Eastern medicine meditations C. high doses of medicines thought to be "hot." D. foods that are "cold."

D. foods that are "cold."

When performing a lung assessment, what technique should the nurse always use first? A. shallow breathing B. normal lung tissue C. decreased adipose tissue D. increased density of lung tissue.

D. increased density of lung tissue.

Which of the following is a cause of hypoactive bowel sounds? A. laxative use B. diarrhea C. hepatitis D. manipulation of the bowel during surgery

D. manipulation of the bowel during surgery Diminished or absent bowel sounds signal decreased motility due to inflammation as seen with peritonitis, following abdominal surgery, or with late bowel obstruction

Heberden's and Bouchard's nodes are hard and nontender and are associated with which of these diseases afflicting older persons? A. metacarpophalangeal bursitis B. rheumatoid arthrtis C. Dupuytren's contracture D. osteoarthritis

D. osteoarthritis

This deep tendon reflex test evaluates the L2,3, & 4 spinal levels. A. biceps B. triceps C. bracheoradialis D. patellar E. achilles

D. patellar

The male client, age 56, has been diagnosed with early cirrhosis, an irreversible fibrotic liver condition. His muscles are tense and his abdomen is distended. What assessment technique would you use to estimate the liver span or determine hepatomegaly? A. inspection B. auscultation C. Murphy's technique D. percussion E. palpation

D. percussion

The nurse hears when percussing the chest a hollow sound with a loud intensity and a low pitch. Which tone matches this description? A. tympany B. dull C. hyperresonant D. resonance

D. resonance

To conduct the Romberg test, you ask the patient to stand with the feet together and arms at the sides. Then the examiner asks the patient to close the eyes. What should the examiner be doing during this test? A. assess the patient's vibratory sensation in both legs B. assess any problems with coordination C. document there is a positive finding if the patient maintains his balance D. stand near the patient E. hold a gait belt placed on the patient

D. stand near the patient

Which of these is the best definition of pulse pressure? A. a reflection of the stress of the patient B. a reflection of the viscosity of the blood C. a measure of vasoconstriction D. the difference between the systolic and diastolic pressure

D. the difference between the systolic and diastolic pressure

When auscultating the lungs of an adult patient, you note that over the posterior lower lobes you hear low-pitched, soft breath sounds, with inspiration being longer than expiration. What are these sounds? A. bronchovesicular breathe sounds and these are normal in that location B. bronchophony sounds and these are normal in that location C. bronchial breath sounds and these are normal in that location D. vesicular breathe sounds and these are normal in that location

D. vesicular breathe sounds and these are normal in that location

What lymph node is deep, under the sternomastoid muscle?

Deep cervical

The nurse is preparing to percuss the abdomen of a patient. The purpose of the percussion is to assess the __________ of the underlying tissue.

Density

True or False: When palpating lymph nodes in the neck, palpate firmly with one hand on top of the other

False Palpate softly with gentle pressure so as to not push the nodes into the neck muscles

Air passing through narrowed bronchioles would produce which of the following adventitious sounds? A. whispered pectoriloquy B. crackles C. vesicular sounds D. bronchial sounds E. wheezes

E. Wheezes

Deep Tendon Reflex Test: The foot is dorsiflexed by the examiner when the tendon is tapped with the flat side of the hammer. A. biceps B. triceps C. bracheoradialis D. patellar E. achilles

E. achilles

TRUE or FALSE: A child, 18 months old, is brought into the clinic for a health screening. To best assess her height, the provider should measure the arm span.

False

TRUE or FALSE: The nurse is screening the client for diplopia. The nurse should ask, "Do you have any blind spots as you shift your gaze?"

False

The best position for the nurse to auscultate S3 and S4 heart sounds is in supine position with the head of the bed in semi-fowlers. True or False

False

True or False: Breast budding is usually normal in non-Hispanic white girls before age 8.

False

True or False: Men do not need to do self-breast examinations, since cancer in this area is rare.

False

True or False: A patient is asked to stick out her tongue.The tongue with a disorder of this cranial nerve will usually deviate to the unaffected or stronger side of the face. The cranial nerve is being tested is the glossopharyngeal nerve.

False The stronger muscles of the tongue will push it to the weaker or paralyzed side. The test is for the hypoglossal nerve.

True or False: The nurse examining a small child with the diagnosis of Down's syndrome will expect to observe narrow palpebral fissures, a thin upper lip, low nasal bridge and short nose, a flat midface and multiple behavior difficulties, as inattention, poor abstract reasoning, and learning disabilities. The child does not have any cardiac problems, nor abnormalities of the tongue, ears, or eyes.

False This is a description of a child with fetal alcohol spectrum disorder

A pregnant women states, " I know labor will be so painful that I won't be able to stand it. I know it sounds awful, but I really dread going into labor." The nurse responds by saying, "Oh, don't worry. I have been through it and although it is painful there are many medications that decrease the pain." What nontherapeutic technique is the nurse using in this scenario?

False reassurance

TRUE or FALSE: When palpating lymph nodes in the neck, it is best to palpate firmly with one hand on top of the other and palpating one side at a time.

False. Palpate softly with gentle pressure with one hand on both sides simultaneously so as to not push the nodes into the neck muscles.

True or False: To conduct the spinal accessory nerve test, you ask the patient to stand with the feet together and arms at the sides. Then the examiner asks the patient to close the eyes.

False. This is the Romberg test to assess balance. The spinal accessory test shrugs the shoulders against resistance.

True or False: For the breast examination when palpation is performed by the nurse on a client, the woman should be in a sitting position with her arms by her side.

False. SHe should be in a supine position with a small pillow tucked under the side to be palpated and her arm raised above her head.

True or False: The nurse is examining a client with osteoarthritis. The nurse knows that crepitation is a dry, crackling sound due to grating of the ends of damaged bone heard with the diaphragm of a stethoscope.

False. The definition of crepitation is correct, but it is felt when you palpate the temperomandibular or other joints. It occurs when you place your hand over a moving joint. It can be audible but not with a stethoscope

True or False: To elicit the Palen's sign, ask the patient to hold both hands palm to palm while flexing the wrists at a 90 degree angle. Acute flexion of the wrists produces paralysis in the wrists of the patient.

False. The hands are held back to back and symptoms with this maneuver indicate carpal tunnel syndrome with parasthesias

TRUE or FALSE: To determine an accurate and valid blood pressure, determine if there is an ausculatory gap by palpating the radial artery, inflating the cuff till the pulse disappears, and then pumping the cuff to 20-30 mm Hg beyond this former measurement after 2 minutes.

False; The nurse needs to wait 15-30 seconds

TRUE or FALSE: The nurse when assessing the person's physical appearance during the general survey should observe the symmetry of the body parts and the posture if it is straight and erect.

False; not for physical appearance but for body structure

Normal findings are up to 2 cm above the sternal angle when elevated at 30 degrees

Inspection of jugular venous pressure

The nurse inspects the woman's breasts for dimpling and asymmetry. What two positions would the nurse ask her to do to fully assess the breast tissue?

Life both of her arms above the head and push her hands onto her hips or push her two palms together

You are interviewing Mr. A., who is a recent immigrant from Mexico. During the course of the interview, you avoid any eye contact, focus on your computer, look intermitgathtently at your watch, and also sit behind the desk with your back to Mr. A. at a social distance. Mr. A begins to feel uncomfortable with your behaviors. What nonverbal behaviors should be substituted for these inappropriate (underlined) ones?

Make appropriate eye contact that shows acceptance of Mr. A., make occasional facilitating gestures instead, and move within close proximity at a personal distance of Mr. A

The nurse is caring for a client who comes to the clinical reporting enlarged lymph nodes in the back of the ear. The nurse palpates the nodes. What findings should the nurse document?

Mobility, delimitation, size, tenderness, location, shape, and consistency

Abnormal heart sound that sounds like a blowing, swishing sound occurring with turbulent blood flow through the heart valves or great vessels

Murmur

Usually located in the 5th ICS within the MCL when present

Palpation of the apical pulse

What instructions should be given to a client having the ocular fundus or retina examined?

Please keep looking at a point straight ahead on the wall across the room, even though my head will get in the way.

What lymph node is in the posterior triangle along the edge of the trapezius muscle?

Posterior cervical

What lymph node is in front of the ear?

Preauricular

An indicator of heart failure assessed when the provider auscultates the apical pulse at the same time he or she palpates the radial pulse.

Pulse deficit

True or False: During pregnancy, the breasts increase in size, the nipples darken, and the areolae widen and become more nodular.

True.

Identify two ingredients for a successful long term weight loss plan

Regular exercise, low calorie diet, low fat diet, monitoring food intake, monitoring weight

During percussion, which sound would you expect to predominate over normal lung tissue?

Resonance

A low pitched, dull heart sound like distant thunder best heard with the bell of the stethoscope occurring immediately after closure of the semilunar valves when the atrioventricular valves open and blood pours into the ventricles. It can persist when sitting up signifying congestive heart failure.

S3

An African American male client aged 68 with hypertension and diabetes mellitus comes to the clinic with a blood pressure of 200/110. The client has not been taking the prescribed antihypertensive, an angiotensin II receptor blocker. The history reveals that he smokes 2 ppd and has a family history of coronary artery disease. What major risk factors does this client have?

Smoking, DM, age, gender, and family history (all but dyslipidemia)

What lymph node is halfway between the angle and the tip of the mandible?

Submandibular

What lymph node overlies the sternomastoid muscle?

Superficial cervical

What lymph node is just above and behind the clavicle, at the sternomastoid muscle?

Supraclavicular

The nurse is documenting that an anterior cervical lymph node is enlarged. The nurse documents that the enlarged node is located in the right arm, is 2 x 1 x 1 mm in size, is dome shaped, is discrete, moveable and not fixed to the underlying muscle, and is soft. Which parameter has the nurse omitted in this description?

Tenderness or pain

The nurse is documenting that an anterior cervical lymph node is enlarged. The nurse documents that the enlarged node is located in the right arm, is 2 x 1 x 1 mm in size, is dome shaped, is discrete, moveable, and not fixed to the underlying muscle, and is soft. Which parameter has the nurse omitted in this description?

Tenderness or pain

How do you test the acoustic cranial nerve?

Test hearing

Which of the following describes normal changes in the respiratory system of the older adult? A. The lungs are less elastic and distensible, decreasing their ability to collapse and recoil. B. Respiratory muscle strength increases to compensate for a decreased vital capacity C. Severe dyspnea is experienced on exertion, due to changes in the lungs. D. There is a decrease in small airway closure, leading to problems with atelectasis

The lungs are less elastic and distensible, decreasing their ability to collapse and recoil.

In the health care setting, the nurse is cautioned against using "why" questions. Give a rationale for this practice. Rephrase this question to a patient who has just come into the emergency room with alcohol on his breath: "Why do you drink so much?" to make it a therapeutic question.

The question can be misconstrued as whining, accusatory, and judgmental. A better way to ask the question would be: "I see you have been drinking. How much alcohol have you consumed?" You might also give the CAGE test.

What are two reasons that the diaphragmatic excursion assessment is no longer done?

The technique of measuring diaphragmatic excursion using percussion is no longer recommended for two reasons: (1) in persons with lung disease, evidence shows that clinicians usually overestimate diaphragmatic movement and that their results differ from chest image by 1 to 3 cm; and (2) evidence shows that diaphragmatic excursion of <2 cm is an unreliable and infrequent sign of COPD.

What gland straddles the trachea in the middle of the neck?

Thyroid gland

What lymph node is under the angle of the mandible?

Tonsillar or jugulodigastric

A child with a deficiency of vitamin D and calcium may likely result in rickets. True or False

True

Persons from the Islamic culture may not drink alcoholic beverages or pork products. True or False

True

Second hand smoke is considered a significant risk factor for coronary artery disease as well as cancer in adults. True or False?

True

TRUE or FALSE: A linear crack in the skin is a fissure.

True

TRUE or FALSE: Gluten free diets may result in deficiencies in fiber, vitamin D, Vitamin B12, folate, iron, zinc, calcium and magnesium.

True

The chest cage commonly shows an increased AP diameter, giving a round barrel shape and kyphosis or an outward curvature of the thoracic spine. True or False

True

True or False: Documentation of key points of the history of an older person who has been the victim of abuse needs to be verbatim using the exact terms used to describe the behaviors of the perpetrator

True

True or False: If a lymph node in the neck that is within normal limits, the practitioner should assess and document that the node feels movable, discrete, soft, and nontender

True

True or False: If a thyroid is enlarged, you should ausculate for the presence of a bruit

True

True or False: Normocephalic correctly describes the normal, round, symmetric skull that is appropriately related to body size.

True

True or False: Reflexes are involuntary and operate below the level of consciousness.

True

True or False: The Abuse Assessment Screen needs to be followed up with open ended questions regarding the specifics acts of abuse

True

True or False: The Danger assessment begins with a calendar so that women can report accurately the frequency and severity of the violent assaults.

True

True or False: The nurse charts that the trachea is midline. This is a normal finding and the health care team should not anticipate any further treatment.

True

True or False: The nurse should place a glass of water by the bedside of the client having a thyroid examination

True

True or False: The temporal artery can be palpated above the zygomatic bone and between the eye and the top of the ear

True

True or False: The thyroid gland may be palpable normally during pregnancy

True

True or False: Vertebra prominens is the vertebra that is palpable when the head is flexed

True

State how to correct this procedure: When assessing a person who is diagnosed with diabetes mellitus for peripheral neuropathy, the nurse touches the monofilament to the bottom of each foot in six places for one to two minutes. The filament should touch the foot gently and the filament should remain straight. The nurse also asks the person to say now or yes when the filament is felt at each of these sites.

When assessing a person who is diagnosed with diabetes mellitus for peripheral neuropathy, the nurse touches the monofilament to the bottom of each foot in ten places for 1-2 seconds. The filament should touch the foot with enough pressure to make the filament bend. The nurse also asks the person to say now or yes when the filament is felt at each of these sites.

A nurse is assessing a client following a motor vehicle accident. The nurse is assessing the client's level of consciousness using the Glasgow coma scale. Which 3 areas should be assessed? Select all that apply. a. motor response to a command b. eye opening response c. vital signs d. deep tendon reflexes e. verbal response to questions

a. motor response to a command b. eye opening response e. verbal response to questions

Which of these questions would be important to ask the client if the client has an aura prior to the tonic-clonic seizure? a. After the seizure, do you spend time sleeping or being confused? b. Does anything bring on the seizure? c. Do you have any warning signs when the seizure starts? d. Where in the body does the seizure begin?

c. Do you have any warning signs when the seizure starts?

Percussion over an area of atelectasis in the lungs would most likely reveal which of these sounds? A. resonance B. hyperresonance C. tympany D. dullness

dullness

Which of the following conditions can result if childhood obesity continues till adulthood? hypertension dyslipidemia type 2 diabetes esophageal cancer osteomalachia

hypertension dyslipidemia type 2 diabetes

Prior to examining the ear of the adult with an otoscope, how should the canal be straightened or positioned?

pulling the auricle up and back

The male client, age 63, had a colon resection 3 days ago. He is very thin. He is NPO and has a nasogastric tube connected to low suction. You begin his abdominal assessment with inspection. What two MOVEMENTS should you look for when inspecting the abdomen?

pulsations and peristalsis

What is the name of the structures in the inner canthus of the eye that drain the fluid providing lubrication produced by the lacrimal gland located in the upper outer corner of the eye?

puncta

How do you test the occulomotor, trochlear, & abducens cranial nerve?

pupillary accomodation downward eye movement tested with extraocular muscle test lateral eye movement tested with extraocular eye test

How do you test the hypoglossal cranial nerve?

range of motion of tongue

How do you test the olfactory cranial nerve?

smell coffee

How do you test the glossopharyngeal and vagus cranial nerves?

test gag reflex and ability to swallow assess the quality of the patient's voice

During the history, the client tells the nurse that " I feel lightheaded and it feels like the room is spinning around me." What medical term would the nurse use to best document this symptom?

vertigo

How do you test the optic cranial nerve?

visual acuity using snellen eye chart

What test would the nurse recommend, especially for African Americans and Hispanic persons between the ages of 45 to 65, to prevent glaucoma and later blindness?

visual field test or confrontation


Kaugnay na mga set ng pag-aaral

Content Area: Med-Surg: Gastrointestinal System

View Set

Matching Scientific Name and Common Name

View Set

TREC Promulgated Contract Forms #351 Final

View Set